The difference of a number and 4/ 3 is 11/3. what is the number

Answers

Answer 1

Answer:

5

Step-by-step explanation:

15/3 - 4/3 = 11/3

15/3 = 5

Answer 2

The difference of a number 5 and 4/ 3 is 11/3.

What is subtraction of fractions?

Subtracting fractions include the subtraction of two or more fractions with the same or different denominators. Like fractions can be subtracted directly but for unlike fractions we need to make the denominators same first and then subtract them.

Given that, the difference of a number and 4/ 3 is 11/3.

Let the unknown number be x.

Here, x-4/3 =11/3

x= 11/3+4/3 (Transpose 4/3 to RHS of the equation)

To add like fractions, add the numerators and keep the denominator same. That is

x= 15/3

x=5

Therefore, the unknown number is 5.

To learn more about the subtraction of fractions visit:

https://brainly.com/question/5220474.

#SPJ2


Related Questions


Han creates a scatter plot that displays the relationship between the number of items sold, x, and the total revenue, y, in dollars.
Han creates a line of best fit and finds that the residual for the point (12, 1,000) is 75. The point (13, 930) has a residual of -40.
Interpret the meaning of -40 in the context of the problem.

Answers

Answer:

In context of the problem, -40 means that 40 was the difference between the actual data, and the estimated data.

Step-by-step explanation:

Here -40 represent the difference between the actual data and the estimated data.

Given that,

Han creates a line of best fit and finds that the residual for the point (12, 1,000) is 75. The point (13, 930) has a residual of -40.

Based on the above information we can say that the -40 shows the difference of data.

Learn more: brainly.com/question/17429689

Salesclerks at a store are giving away candles. There are 12 candles on a tray. Three are white and two are red. The rest are green or blue. When choosing a candle at random from the tray, a customer is as likely as not to select a green candle. What tis the likelihood that the customer will select a blue candle? Tell whether it is impossible, unlikely, equally likely as not or certain. Explain your answer.

Answers

Answer:

one blue candle. 1/12 probability.

Step-by-step explanation:

if they are as likely as not to select a green candle, then there are 6 green ones. so now we have accounted for 11 if I am right, so only one can be blue

It is equally likely as not that the customer will select a blue candle.

What is probability?

It is the chance of an event to occur from a total number of outcomes.

The formula for probability is given as:

Probability = Number of required events / Total number of outcomes.

Example:

The probability of getting a head in tossing a coin.

P(H) = 1/2

We have,

Let the number of green candles on the tray be G, and the number of blue candles be B.

We know that there are 12 candles in total on the tray, so:

G + B = 12 (total number of green and blue candles is 12)

G/(G+B) = 1/2 (as the probability of selecting a green candle is equal to the probability of selecting a non-green candle)

We can use these two equations to solve for G and B:

G + B = 12

G = (12 - B)

Substituting into the second equation:

(12 - B) / (12 - B + B) = 1/2

(12 - B) / 12 = 1/2

12 - B = 6

B = 6

So there are 6 blue candles on the tray.

Since there are 12 candles in total, and 6 of them are blue, the probability of selecting a blue candle at random is:

P(selecting blue) = B / (G + B) = 6 / 12 = 1/2

Therefore,

It is equally likely as not that the customer will select a blue candle.

This means that the probability of selecting a blue candle is 50-50, which is neither impossible nor certain, but rather equally likely as selecting a green candle.

Learn more about probability here:

https://brainly.com/question/14099682

#SPJ3

Please give me the correct answer.Only answer if you're very good at math.​

Answers

Answer:

45m - 900 = 90m

Answer:

First box 45, second box 900, third box 90

Step-by-step explanation:

0.45 x 100 = 45

9 x 100 = 900

0.9 x 100 = 90

HELP I NEED HELP ASAP HELP I NEED HELP ASAP HELP I NEED HELP ASAP HELP I NEED HELP ASAP
HELP I NEED HELP ASAP HELP I NEED HELP ASAP HELP I NEED HELP ASAP HELP I NEED HELP ASAP


The cost of renting ice skates at a skating is $1.25. The rink charges $3.25 for each hour, x, of skating. Which graph best models this situation, where y is the total cost to go skating?

Answers

Answer:

Step-by-step explanation:

y = 3.25x + 1.25

1.25 is y-intercept

3.25 is the slope of a graph.

If x = 0, then y = 1.25

(0, 1.25)

If x = 3, then y = 3.25(3) + 1.25 = 11

(3, 11)

It should be (3,11) goodluck

NEED Help ASAP I Will GIVE BRAINLIST

What is the measure, in degrees, of angle x?
A
2.3

B
42.7

C
47.3

D
57.3

Answers

57.3 is the answer dididjwkjakz

In right triangle angles L and M are complementary find the measure of angle L

Answers

Answer and Step-by-step explanation:

90 - 51 = 39.

Angle L is equal to 39, since angle M is equal to 51.

The angles are complementary, so they add up to 90, which is why we subtract 51 from 90 to get 39.

Have a great day!

#teamtrees #PAW (Plant And Water)

is this a function?

Answers

Answer:

No and yes. it depends on what type of function you are looking for this will be a Linear Function.

please help
i need to complete this by 5:00pm

Answers

Answer:

A) 6

Step-by-step explanation:

6 is 8 points away from -2

0 is 2 points away from -2

-1 is 1 point away from -2

-9 is 7 points away from -2

Brainliest ,please help, Line L has the equation y = 5x + 12 Write down the gradient of line L

Answers

In the equation y=mx+c m= the gradient of the line and our m=5 so gradient of the line is 5

The gradient of line L is 5

What is the gradient f the line?"It is the ratio of the the change in the 'y' coordinate with respect to the change in the 'x' coordinate of that line.""It is used to calculate the steepness of a line."What is the slope-intercept form of the line?

"y = mx + c, where m is the slope and c is the y-intercept."

For given question,

We have been given an equation of the line L  y = 5x + 12

Comparing above equation with the slope-intercept form of the line,

we have m = 5 and c = 12

Here, slope of the line L is 5

We know that the slope of the line is the  the change in the y coordinate with respect to the change in the x coordinate of the line.

This means, the gradient of the line L is 5

Therefore, the gradient of line L is 5

Learn more about the gradient of the line here:

https://brainly.com/question/11693181

#SPJ2

Find the value of a and b when x = 10
a =
5x2
2
b=
2x2(x - 5)
10x
help

Answers

Answer: x-15=0 is x-15

x-4=0 is x=4

Hope this helps :)

Step-by-step explanation:

Solve for R*
6r - 4 = 32
please help

Answers

Answer:

r is 6

Step-by-step explanation:

This is because 6x6=36 and 36-4=32

Q2. 1 a) Dave wants to invest $2000 in an account that earns 2.5% interest compounded monthly. If he
leaves the money in the account for 5 years, how much money will he have at the end of those 5 years?

Answers

Question: Dave wants to invest $2000 in an account that earns 2.5% interest compounded monthly. If he  leaves the money in the account for 5 years, how much money will he have at the end of those 5 years?

Answer:

Step-by-step explanation:

Write the integer represented by each point on the number line shown below. plsss helpp ME​

Answers

You need to provide the number line

Last January, the average cost of a one-night stay at a hotel was $132.52. Since then, the hotel has changed ownership, and
several changes have been made to the cost of staying at the hotel.
• Last summer, the new owners adjusted room prices so that the average cost of a one-night stay increased by $34.65.
Last December, the new owners offered a $57.80 per night discount to guests who belong to their loyalty club.
. Last week, the new owners introduced a service fee of $4.95 for any reservation made over the phone.
How much should a customer expect to pay for a one-night stay if they are a member of the hotel's loyalty club and make
their reservation over the phone?

Answers

Answer:

$114.32

Step-by-step explanation:

Original Price: $132.52

increased: $34.65

Loyalty Club: -$57.80

Phone Fee: $4.95

You add all of them together, except for the Loyalty Club Fee. You subtract that fee by the total.

The amount of pay for a one-night stay, if they are a member of the hotel's loyalty club and make their reservation over the phone, will be $113.77

What will be the amount?

The average cost of a one-night stay at a hotel was $132.52. Since then, the hotel has changed ownership, and several changes have been made to the cost of staying at the hotel. Last summer, the new owners adjusted room prices so that the average cost of a one-night stay increased by $34.65.

Last December, the new owners offered a $57.80 per night discount to guests who belong to their loyalty club. Last week, the new owners introduced a service fee of $4.95 for any reservation made over the phone.

The price will be calculated as:-

Original Price: $132.52

increased:   $34.65

Loyalty Club : - $57.80

Phone Fee: $4.95

You add all of them together, except for the Loyalty Club Fee. You subtract that fee by the total.

=  132.52  +   34.65   +   4.95   -   57.80  =  $113.77

Therefore the amount of pay for a one-night stay, if they are a member of the hotel's loyalty club and make their reservation over the phone, will be $113.77.

To know more about the amount follow

https://brainly.com/question/25109150

#SPJ2

a polynomial contains number of terms​

Answers

Correct question is;

A polynomial contains _____ number of terms

Answer:

(n + 1)

Where n is the degree of the polynomial

Step-by-step explanation:

Now, for a polynomial that has degree of n = 1, the number of terms it will have is 2.

Similarly, a polynomial that has degree of n = 2, the number of terms it will have is 3.

This means from the above, we can deduce that the number of terms in a polynomial with degree "n" is n + 1.

We can therefore conclude that a polynomial can have any finite number of terms as n + 1

Tickets to a movie cost $5 for adults and $3 for students. A group of friends purchased 18 tickets for $82.00. How many adults ticket did they buy? *

4
14
12
10

Answers

14, because 14 times 5 equals 70, and then you add 4 children tickets so 70 plus 12 equals 82$

Please help

How much per song?

Answers

Answer:

1.20 per song

Step-by-step explanation:

Its quite simple. just take 6 divided by 5

1.20 per person is the answer

Please help me this is my final!!!

Answers

Answer:

it is a. 43 sides

Step-by-step explanation:

(n-2)*180=number of degrees in a polygon

Answer:

43

Step-by-step explanation:

super straight is what?

a. fasle

b. true

c. transphobic

d. all the above

e. none of them are

Answers

Answer:

C C C C C C C C C C C C C c ccccccc

Answer:

c and a

Step-by-step explanation:

it is transphobic and they are d    um   b <3

its not real theyre just st           u         pi   d^^

I hope this helped <3

Write each phrase as an algebraic expression.

four times a number

a number divided by 14

Answers

Answer:

#1. Fourteen divided by a number.

#2. Four times a number.

#3. A number divided by 14.

#4. A number minus 14.

Step-by-step explanation:

I hope this helps., I just need more points. So sorry.

Explanation:
"the quotient" means to divide.
"the quotient of 14" means 14 is going to be divided by something. So we can write:
14
÷
The something it will be divided by is "the difference between a number and -7".
Lets call "a number"
n
.
"the difference" means we are going to subtract something from
n
and what we are going to subtract is a

7
. We can write this as:
n

(

7
)
Putting this together gives:
14
÷
(
n

(

7
)
)
Or
14
n

(

7
)

I have 5 mins to get this done.

Answers

1.)A.74
2.)B.40
3.)A.92

Hope this helps

Have a great day/night

Feel free to ask any questions

The sum of two numbers is 42. One number is 2 times as large as the other. What are the numbers?
Larger number:
Smaller number:

Answers

Answer:

larger number = 28

smaller number = 14

Step-by-step explanation:

42/3 = 14

14 x 2 = 28

to check ur answer u could add them together so they equal 42

14 + 28 = 42

Answer:

Larger number : 28

Smaller number : 14

Step-by-step explanation:

a + b = 42

a = 2b

2b + b = 42

3b = 42 (divide both sides by 3)

b = 14

a = 2 x 14

a = 28


Taryn's dog, Skeeter, consumes an average of 1,155 calories per day. However, Skeeter participated in a dog race this morning, so
he needs more than his average number of calories. Skeeter has already eaten 693 calories today, and Taryn feeds him
231 calories per meal.
2313 + 693 > 1,155
How many more meals does Taryn need to feed Skeeter today? Select the inequality that includes the smallest number of meals
Taryn can feed Skeeter while still feeding him enough calories for the day.
A. x>3
B. x>231
C. x>2
D. x > 693

Answers

Answer:

The answer is C. x = 2

What number could replace m below?
4/12 = 1/m

Answers

Answer:

3

Step-by-step explanation:

4/12 = 1/3 yepppp djdkkdkdd

Solve for X
Potential answers are (WILL GIVE BRAINLIEST
1. 0
2. 11.4
3. 16
4. -16

Answers

Answer:

3)  16

Step-by-step explanation:

these are alternate-interior angles which are congruent

10x-23 = 137

10x = 160

x = 16

PIease help me to find this answer

Answers

Answer:

870

Step-by-step explanation:

1035-165=870

the answer is 870.

a little help plssss

Answers

Answer:

26 This is perimeter

Step-by-step explanation:

Perimeter= Add up all the outside numbers

Area= Length x width

The distance around 2 bicycle wheel is 22.98ft. What is the diameter
A. 7.3
B. 3.2
C. 1.3
D. 10.3
Please help!!!!!! i need the correct answer
I will mark you brainliest.

Answers

Id say 7.3. Hope its correct.
It’s A I agree with them at the top

ALGEBRA 1:
FIND THE PRODUCT OF THE FOLLOWING:
(2x+1)^2
please somebody help me, if you’re correct i’ll give you an extra 50 points for helping. please do not use my question for points as i am failing and truly need help i’m so stressed lol.

Answers

(2x+1)^2
=(2x+1)(2x+1)
=4x^2+2x+2x+1
=4x^2+4x+1

Answer:

Step-by-step explanation:

(2x-1)(2x-1)  Work outside then inside

4x^2 and +1 are the outside terms

-2x and -2x are the inside terms

so the answer will be

4x^2 - 4x +1

I need help with this, I’ll give brainliest!

Answers

Answer:

A

Step-by-step explanation:

Only graph A satisfies the domains provided

Other Questions
Drag and drop the description to match each person into the boxes. Lord CornwallisNathanael GreeneAlexander HamiltonWilliam HoweMartha WashingtonGeorge Rogers Clark .Frontiersman who fought British and their Indian allies in Ohio valley. .Commander of British ship Serapis and said "I have not yet begun to fight." .Quartermaster general of the Continental Army; brought caches of food and supplies to Valley Forge and led troops in the South. .Commander of all British forces in America until 1778. .General's wife who helped boost morale at Valley Forge. .American colonel and aide to Washington who captured a key British earth fortress at Yorktown. pls help 50 points!!!! What role do the four kinds of taste buds play in our survival? help me who know deku 12) Symbols t leche en el desayuno? cot^2x-csc^2x= please include the steps thank you !!will give branliest What is the value of y in the following equation? -10y + 4(3y 8) = -64 I just want the coordinates, thanks :) what is 4 and half hours in minutes Please help ASAP ASAP ASAP asap As a result of the Battle of Coral Sea, U.S.forces successfullyA captured Japan's largest aircraft carrier.B. regained control of the Philippines.C. prevented Japan from invading New Guinea.D.sunk two ships in Japan's fleet of ships. . The source above describes the philosophy of the Renaissance called..A. PredestinationB. Natural LawC. Neo-ConfucianismD. Humanism What else could I say????? What might you have to do in order to understand someones cryptic remarks? (Giving brainliest)! 1/2 of 2/2 = ??????????? How did the Great Depression affect Latin America? a full glass of water can hold 1/6 of a bottle how many of glasses can be filled with 3 1/2 of bottles of water Samuel invests 45 500 in an account that pays 4.5% compound interest per annum for the five years, then 3.5% . Work out how much money Samuel will have in his account in 13 years. Which organism is the Least related to humans? Explain I need help with this question please Solve:One week, seventy-two percent of the animals that came to a veterinary hospital were dogs. If there were 230 animals total, how many were dogs?